You are on page 1of 21

ART VII SEC 13

FLORES VS DRILON,
The constitutionality of Sec. 13, par. (d), of R.A.
7227, otherwise known as the "Bases Conversion and
Development Act of 1992," under which respondent Mayor
Richard J. Gordon of Olongapo City was appointed Chairman
and Chief Executive Officer of the Subic Bay Metropolitan
Authority (SBMA), is challenged in this original petition with
prayer for prohibition, preliminary injunction and temporary
restraining order "to prevent useless and unnecessary
expenditures of public funds by way of salaries and other
operational expenses attached to the office . . . ." Paragraph (d)
reads
(d) Chairman administrator The President
shall appoint a professional manager as
administrator of the Subic Authority with a
compensation to be determined by the Board
subject to the approval of the Secretary of
Budget, who shall be the ex oficio chairman
of the Board and who shall serve as the chief
executive officer of the Subic
Authority: Provided, however, That for the
first year of its operations from the effectivity
of this Act, the mayor of the City of Olongapo
shall be appointed as the chairman and chief
executive officer of the Subic
Authority (emphasis supplied).
Petitioners, who claim to be taxpayers, employees of the U.S.
Facility at the Subic, Zambales, and officers and members of
the Filipino Civilian Employees Association in U.S. Facilities in
the Philippines, maintain that the proviso in par. (d) of Sec. 13
herein-above quoted in italics infringes on the following
constitutional and statutory provisions: (a) Sec. 7, first par., Art.
IX-B, of the Constitution, which states that "[n]o elective official
shall be eligible for appointment or designation in any capacity
to any public officer or position during his tenure," because the
City Mayor of Olongapo City is an elective official and the
subject posts are public offices; (b) Sec. 16, Art. VII, of the
Constitution, which provides that "[t]he President shall . . . .
appoint all other officers of the Government whose
appointments are not otherwise provided for by law, and those
whom he may be authorized by law to appoint", since it was
Congress through the questioned proviso and not the
President who appointed the Mayor to the subject posts;
ISSUE: Whether or not the proviso in Sec. 13, par. (d), of
R.A. 7227 violates the constitutional proscription against
appointment or designation of elective officials to other
government posts
HELD: YES. The rule expresses the policy against the
concentration of several public positions in one person, so that
a public officer or employee may serve full-time with dedication
and thus be efficient in the delivery of public services. It is an
affirmation that a public office is a full-time job. Hence, a public
officer or employee, like the head of an executive department
described in Civil Liberties Union v. Executive Secretary, and
Anti-Graft League of the Philippines, Inc. v. Philip Ella C. Juico,
as Secretary of Agrarian Reform should be allowed to attend to
his duties and responsibilities without the distraction of other
governmental duties or employment. He should be precluded
from dissipating his efforts, attention and energy among too
many positions of responsibility, which may result in
haphazardness and inefficiency.
In this case, the subject proviso directs the President
to appoint an elective official, i.e., the Mayor of Olongapo City,

to other government posts (as Chairman of the Board and


Chief Executive Officer of SBMA).
In any case, the view that an elective official may be
appointed to another post if allowed by law or by the primary
functions of his office ignores the clear-cut difference in the
wording of the two (2) paragraphs of Sec. 7, Art. IX-B, of
the Constitution. While the second paragraph authorizes
holding of multiple offices by an appointive official when
allowed by law or by the primary functions of his position, the
first paragraph appears to be more stringent by not providing
any exception to the rule against appointment or designation of
an elective official to the government post, except as are
particularly recognized in the Constitution itself, e.g., the
President as head of the economic and planning agency;
the Vice-President, who may be appointed Member of the
Cabinet; and, a member of Congress who may be
designated ex officio member of the Judicial and Bar Council.
It is further argued that the SBMA posts are merely ex
officio to the position of Mayor of Olongapo City, hence, an
excepted circumstance, citing Civil Liberties Union v. Executive
Secretary. However, the court held that the Congress did not
contemplate making the subject SBMA posts as ex officio or
automatically attached to the Office of the Mayor of Olongapo
City without need of appointment. The phrase "shall be
appointed" unquestionably shows the intent to make the SBMA
posts appointive and not merely adjunct to the post of Mayor of
Olongapo City. Had it been the legislative intent to make the
subject positions ex officio, Congress would have, at least,
avoided the word "appointed" and, instead, "ex officio"
would have been used. Even in the Senate deliberations, the
Senators were fully aware that subject proviso may contravene
Sec. 7, first par., Art. IX-B, but they nevertheless passed the bill
and decided to have the controversy resolved by the courts.
WHEREFORE, the proviso in par. (d), Sec. 13, of R.A. 7227,
which states: ". . . Provided, however, That for the first year of
its operations from the effectivity of this Act, the Mayor of the
City of Olongapo shall be appointed as the chairman and chief
executive officer of the Subic Authority," is declared
unconstitutional; consequently, the appointment pursuant
thereto of the Mayor of Olongapo City, respondent Richard J.
Gordon, is INVALID, hence NULL and VOID.
BITONIO V COA
Instant petition filed under Rule 64 of the Revised Rules of
Court seeks the annulment of the decision of COA denying the
petitioners motion for reconsideration of the COA Notices of
Disallowance.
Benedicto Ernesto R. Bitonio, Jr was appointed Director IV of
the Bureau of Labor Relations in the DOLE. Acting Secretary
Jose S. Brillantes of DOLE designated Bitonio to be the DOLE
representative to the Board directors of PEZA. After post audit
of PEZAs disbursement transactions, COA disallowed the per
diem of Bitonio.
Arguments of COA:
1.
Cabinet members, their deputies and assistants holding
other offices in addition to primary office are not allowed to
receive compensation to subsequent office
Argument of Bitonio:
Rank equivalent to a Secretary, Undersecretary or Assistant
Secretary and other appointive officials below the rank of
Assistant secretary are not covered by the prohibition
W/N Executive Order No. 284 allowing government officials to
hold multiple positions in government, unconstitutional
RULING: Petition DENIED

Main point: Cabinet Secretaries, Undersecretaries, and their


Assistant Secretaries, are prohibited to hold other government
offices or positions in addition to their primary positions and to
receive compensation therefor, except in cases where the
Constitution expressly provides.
PUBLIC INTEREST GRP V ELMA
Respondent Elma was appointed as Chairman of ThePresident
ialCommission on Good Government (PCGG) ON 30 October
1998. Thereafter, during his tenure as PCGG Chairman, he
was appointed as Chief Presidential legal counsel (CPLC).
He accepted the second appointment, but waived any
remuneration
that
he
may
receive
as CPLC. Petitioners sought to have both appointments
declared as unconstitutional and therefore, null and void.
W/N the said appointment is unconstitutional
RULING: Petition GRANTED
Main point: The prohibition in Section 13, Article VII of the 1987
Constitution does not apply to respondent Elma since neither
the PCGG Chairman nor the CPLC is a Cabinet secretary,
undersecretary, or assistant secretary. Even if this Court
assumes, arguendo, that Section 13, Article VII is applicable to
respondent Elma, he still could not be appointed concurrently
to the offices of the PCGG Chairman and CPLC
because neither office was occupied by him in an exofficio capacity, nor the primary functions of one office do not
require an appointment to the other post. Moreover, even if the
appointments in question are not covered by Section 13, Article
VII of the 1987 Constitution, said appointments are still
prohibited under Section 7, Article IX-B, which covers all
appointive and elective officials, due to the incompatibility
between the primary functions of the offices of the PCGG
Chairman and the CPLC.
Section 13. Prohibitions
DELA CRUZ V CO
FACTS
Petitioners, who are alternates of the
Executive Secretaries, sitting as members of the
Board of Directors of the National Housing
Authority (NHA) from 1991 to 1996 challenged the
COA issued Memorandum of Notice of Disallowance
to
heads/auditors/team leaders of the national
government agencies and government-owned and
controlled corporations which have effected payment
of any form of additional compensation or
remuneration to cabinet secretaries, their deputies
and assistants, or their representatives, in violation of
the rule on multiple positions to (a) immediately
cause the disallowance of such additional
compensation or remuneration given to and received
by the concerned officials, and (b) effect the refund of
the same.
The petitioners contend that the constitutional
ban against dual or multiple positions applies only to
the members of the Cabinet, their deputies or
assistants. It does not cover other appointive officials
with equivalent rank or those lower than the position
of Assistant Secretary which in their case, the NHA
Directors are not Secretaries, Undersecretaries or
Assistant Secretaries and that they occupy
positions lower than the position of Assistant
Secretary.
ISSUE

RULING

Whether or not the ALTERNATES OF


EXECUTIVE SECRETARIES are entitled for
additional compensation
No. Since the Executive Department Secretaries,
as ex-officio members of the NHA Board, are
prohibited from receiving extra (additional)
compensation, whether it be in the form of a per diem
or an honorarium or an allowance, or some other
such euphemism," it follows that petitioners who sit as
their alternates cannot likewise be entitled to receive
such compensation. A contrary rule would give
petitioners a better right than their principals.
Note:
*The ex-officio position being actually and in legal
contemplation part of the principal office, it follows
that the official concerned has no right to receive
additional compensation for his services in the said
position. The reason is that these services are already
paid for and covered by the compensation attached to
his principal office.
*The acts of the alternates shall be considered the
acts of their principal

Funa v Ermita
FACTS:
On October 4, 2006, President Gloria MacapagalArroyo appointed respondent Maria Elena H. Bautista
(Bautista) as Undersecretary of the Department of
Transportation and Communications (DOTC).
On September 1, 2008, following the resignation of
then MARINA Administrator Vicente T. Suazo, Jr.,
Bautista was designated as Officer-in-Charge (OIC),
Office of the Administrator, MARINA, in concurrent
capacity as DOTC Undersecretary.
During the pendency of this petition, Bautista was
appointed Administrator of the MARINA and she
assumed her duties and responsibilities as such
on February 2, 2009.
Dennis A. B. Funa in his capacity as taxpayer,
concerned citizen and lawyer, filed the instant petition
challenging the constitutionality of Bautistas
appointment/designation, which is proscribed by the
prohibition on the President, Vice-President, the
Members of the Cabinet, and their deputies and
assistants to hold any other office or employment.
ISSUE:
Whether or not the designation of
respondent Undersecretary Maria Elena H. Bautista
as Officer-in-Charge (OIC) of the Maritime Industry
Authority (MARINA) in concurrent capacity as DOTC
Undersecretary valid
RULING:
No. It must be stressed though that while the
designation was in the nature of an acting and
temporary capacity, the words hold the office were
employed. Such holding of office pertains to both
appointment and designation because the appointee or
designate performs the duties and functions of the
office. The 1987 Constitution in prohibiting dual or
multiple offices, as well as incompatible offices, refers to
the holding of the office, and not to the nature of the
appointment or designation.
Respondent Bautista being then the appointed
Undersecretary of DOTC was thus covered by
the stricter prohibition under Section 13, Article VII.
She also failed to demonstrate that her designation as

OIC of MARINA was an ex-officio capacity as required


by the primary functions of her office as DOTC
Undersecretary for Maritime Transport.
Given the vast responsibilities and scope of administration of the
Authority, we are hardly persuaded by respondents submission
that respondent Bautistas designation as OIC of MARINA was
merely an imposition of additional duties related to her primary
position as DOTC Undersecretary for Maritime Transport. It
appears that the DOTC Undersecretary for Maritime Transport is
not even a member of the Maritime Industry Board
Other Prohibitions
DOROMAL V SANDIGANBAYAN
FACTS:That in or about the period from April 28, 19866 to
October 16, 1987, in Metro Manila, Philippines and within the
jurisdiction of this Honorable Court, the above-named
accused, a public officer, being then Commissioner of the
Presidential Commission on Good Government, did then and
there willfully and unlawfullyhave direct or indirect financial
interest in the Doromal International Trading Corporation, an
entity which transacted or entered into a business transaction
or contract with the Department of Education, Culture and
Sports and the National Manpower and Youth Council, both
agencies of the government which business, contracts or
transactions he is prohibited by law and the constitution from
having any interest.
On August 22, 1988, the Special Prosecutor filed a "Motion to
Suspend Accused Pendente Lite" pursuant to Section 13 of the
Anti- Graft and Corrupt Practices Act (R.A. 3019). Over the
petitioner's objection (because the President had earlier
approved his application for indefinite leave of absence as
PCGG commissioner "effective immediately and until final
decision of the courts in your case" [Annex S-1, p. 189,
Rollo]), the Sandiganbayan on September 5, 1988 ordered his
suspension pendente lite from his position as PCGG
Commissioner and from any other office he may be
holding(Annex T). His motion for reconsideration of that
order was also denied by the Court (Annex Y).
ISSUE:Hence, this petition for certiorari and prohibition
alleging that the Sandiganbayan gravely abused its discretion:
(1) in denying the petitioner's motion to quash the
information in Criminal Case No. 12893; and,
(2) in suspending the petitioner from office despite the
President's having previously approved his indefinite leave of
absence " until final decision" in this case.
RULING:
The petitioner contends that as the preliminary investigation
that was conducted prior to the filing of the original
information in Criminal Case No. 12766 was nullified by this
Court, another preliminary investigation should have been
conducted before the new information in Criminal Case No.
12893 was filed against him. The denial of his right to such
investigation allegedly violates his right to due process and
constitutes a ground to quash the information.
The public respondent argues that another preliminary
investigation is unnecessary because both old and new
informations involve the same subject matter a violation of
Section 3 (H) of R.A. No. 3019 (the Anti-Graft and Corrupt
Practices Act) in relation to Section 13, Article VII of the 1987
Constitution. Moreover, the petitioner allegedly waived the
second preliminary investigation by his failure to comply with
the Court's Order dated August 12, 1988 directing him to
submit a statement of new or additional facts, duly supported

by photo copies of documents which he would present should


a new preliminary investigation be ordered (Annex H, p. 94,
Rollo).
The petition is meritorious. A new preliminary investigation of
the charge against the petitioner is in order not only because
the first was a nullity but also because the accused demands
it as his right. Moreover, the charge against him had been
changed, as directed by the Ombudsman.
That right of the accused is "a substantial one." Its denial over
his opposition is a "prejudicial error, in that it subjects the
accused to the loss of life, liberty, or property without due
process of law"
However, as the absence of a preliminary investigation is not
a ground to quash the complaint or information (Sec. 3, Rule
117, Rules of Court), the proceedings upon such information
in the Sandiganbayan should be held in abeyance and the
case should be remanded to the office of the Ombudsman for
him or the Special Prosecutor to conduct a preliminary
investigation.
There is no merit in petitioner's insistence that the
information should be quashed because the Special
Prosecutor admitted in the Sandiganbayan that he does not
possess any document signed and/or submitted to the DECS
by the petitioner after he became a PCGG Commissioner (p.
49, Rollo). That admission allegedly belies the averment in
the information that the petitioner "participated' in the
business of the DITC in which he is prohibited by the
Constitution or by law from having any interest.
The Sandiganbayan in its order of August 19, 1988 correctly
observed that "the presence of a signed document bearing
the signature of accused Doromal as part of the application to
bid ... is not a sine qua non"
Since the petitioner is an incumbent public official charged in
a valid information with an offense punishable under the
Constitution and the laws (RA 3019 and PD 807), the law's
command that he "shall be suspended from office"pendente
lite must be obeyed. His approved leave of absence is not a
bar to his preventive suspension for, as indicated by the
Solicitor General, an approved leave, whether it be for a
fixed or indefinite period, may be cancelled or shortened at
will by the incumbent.
Obviously, the Court found the petitioner's preventive
suspension for seven (7) months to be unreasonable for it
would be to countenance a situation where the preventive
suspension can, in effect, be the penalty itself without a
finding of guilt after due hearing.
Since his preventive suspension has exceeded the reasonable
maximum period of ninety (90) days provided in Section 42 of
the Civil Service Decree of the Philippines (P.D. 807), it should
now be lifted.
WHEREFORE, the petition for certiorari and prohibition is
granted. The Sandiganbayan shall immediately remand
Criminal Case No. 12893 to the Office of the Ombudsman for
preliminary investigation and shall hold in abeyance the
proceedings before it pending the result of such
investigation. The preventive suspension of the petitioner is
hereby lifted. No costs.
Section 15: Prohibition on Appointment
IN RE APPOINTMENTS OF VALENZUELA AND VALLARTA
ISSUE:
(1)

whether, during the period of the ban on


appointments imposed by Section 15, Article VII of
the Constitution, the President is nonetheless
required to fill vacancies in the judiciary (lower
courts), in view of Sections 4(1) and 9 of Article VIII

(2)

whether he can make appointments to the judiciary


during the period of the ban in the interest of
public service.

FACTS: Referred to the Court En Banc by the Chief Justice


are the appointments signed by His Excellency the President
under the date of March 30, 1998 of Hon. Mateo A. Valenzuela
and Hon. Placido B. Vallarta as Judges of the Regional Trial
Court of Branch 62, Baguio City and of Branch 24, Cabanatuan
City, respectively. The appointments were received at the
Chief Justice's chambers on May 12, 1998. The referral was
made in view of the serious constitutional issue concerning
said appointments arising from the pertinent antecedents.
The issue was first ventilated at the meeting of the Judicial
and Bar Council on March 9, 1998. The meeting had been
called, according to the Chief Justice asEx OfficioChairman,
to discuss the question raised by some sectors about the
"constitutionality of *** appointments" to the Court of
Appeals, specifically, in light of the forthcoming presidential
elections.
RULING:
The Court further Resolved that
(1) pending the foregoing proceedings and the
deliberation by the Court on the matter, and until
further orders, no action be taken on the
appointments of Hon. Valenzuela and Hon. Vallarta
which in the meantime shall be held in abeyance and
not given any effect and said appointees shall refrain
from taking their oath of office; and that
(2) exercising its power of supervision over the
Judicial and Bar Council, said Council and its ex
officio and regular Members herein mentioned be
INSTRUCTED, as they are hereby INSTRUCTED, to
defer all action on the matter of nominations to fill
up the lone vacancy in the Supreme Court or any
other vacancy until further orders.
On May 12, 1998, the Chief Justice received from Malacaang
the appointments of two (2) Judges of the Regional Trial
Court mentioned above.This places on the Chief Justice the
obligation of acting thereon; i.e., transmitting the
appointments to the appointees so that they might take their
oaths and assume their duties of their office. The trouble is
that in doing so, the Chief Justice runs the risk of acting in a
manner inconsistent with the Constitution, for these
appointments appear prima facie, at least, to be expressly
prohibited by Section 15, Article VII of the charter. This
circumstance, and the referral of the constitutional question
to the Court in virtue of the Resolution of May 8, 1998, supra,
operate to raise a justiciable issue before the Court, an issue
of sufficient importance to warrant consideration and
adjudication on the merits.
The provision of the Constitution material to the inquiry at
bar read as follows
Section 15, Article VII:
"Two months immediately before the next
presidential elections and up to the end of his term, a
President or Acting President shall not make
appointments, except temporary appointments to
execute positions when continued vacancies therein will
prejudice public service or endanger public safety."
Section 4 (1), Article VIII:
"The Supreme Court shall be composed of a Chief
Justice and fourteen Associate Justices. It may sit en
banc or in its discretion, in divisions of three, five, or

seven Members. Any vacancy shall be filled within


ninety days from the occurrence thereof."
Section 9, Article VIII:
"The Members of the Supreme Court and judges in
lower courts shall be appointed by the Presidentfrom
the list of at least three nominees prepared by the
Judicial and Bar Council for every vacancy. Such
appointments need no confirmation.
For the lower courts, the President shall issue the
appointments within ninety days from the submission of
the list."
One interpretation that immediately suggests itself is that
Section 4(1), Article VIII is a general provision while Section
15, Article VII is a particular one; that is to say, normally,
when there are no presidential elections - which after all
occur only every six years - Section 4(1), Article VIII shall
apply: vacancies in the Supreme Court shall be filled within
90 days; but when (as now) there are presidential elections,
the prohibition in Section 15, Article VII comes into play:the
President shall not make any appointments. The reason for
said prohibition, according to Fr. J. Bernas, S.J., an authority
on Constitutional Law and himself a member of the
Constitutional Commission, is "(I)n order not to tie the hands
o f t h e i n c o m i n g Pr e s i d e n t t h r o u g h m i d n i g h t
appointments."Another interpretation is that put forth in the
Minutes of the JBC Meeting of March 9, 1998.
DE LA RAMA V CA
FACTS: Upon his assumption to the position of Mayor of
Pagbilao, Quezon, petitioner Conrado L. de Raffia wrote a
letter dated July 13, 1995 to the Civil Service Commission (or
CSC), seeking the recall of the appointments of fourteen (14)
municipal employees. Petitioner de Raffia justified his recall
request on the allegation that, the appointments of the said
employees were "midnight" appointments of the former
mayor, Ma. Evelyn S. Abeja, done in violation of Article VII,
Section 15 of the 1987 Constitution.
Three of the above-named employees, namely: Elsa Marino,
Morell Ayala, and FlordelizaOriazel, filed with the CSC a claim
for payment of their salaries, alleging that although their
appointments were declared permanent by ConradoGulim,
petitioner de Rama withheld the payment of their salaries
and benefits pursuant to Office Order No. 95-01 wherein the
appointments of the said fourteen (14) employees were
recalled.
Legal and Quasi-Judicial Division of the CSC issued an Order
finding that since the claimants-employees had assumed their
respective positions and performed their duties pursuant to
their appointments, they are therefore entitled to receive
the salaries and benefits appurtenant to their positions.
On April 30, 1996, the CSC denied petitioner's request for the
recall of the appointments of the fourteen employees, for
lack of merit. The CSC also dismissed petitioner's allegation
that these were "midnight" appointments, pointing out that
the Constitutional provision relied upon by petitioner
prohibits only those appointments made by an outgoing
President and cannot be made to apply to local elective
officials. Thus, the CSC opined, "the appointing authority can
validly issue appointments until his term has expired, as long
as the appointee meets the qualification standards for the
position."
After reviewing the facts and issues raised by petitioner, the
Court of Appeals issued a Resolution dated May 16, 1997

which held that there was no abuse of the power of


appointment on the part of the outgoing mayor. The Court of
Appeals further held that the fact that the appointments of
Marino, Ayala, Ramos, Mendoza and Glory were made more
than four (4) months after the publication of the vacancies to
which they were appointed is of no moment.
Petitioner filed a motion for reconsideration arguing that the
appellate court erred in upholding the CSC's resolutions.
ISSUE: WON the appointments made by the outgoing Mayor of
Pagbilao, Quezon violates the Constitutional provision under
Art VII Sec 15.
RULING:The records reveal that when the petitioner brought
the matter of recalling the appointments of the fourteen (14)
private respondents before the CSC, the only reason he cited
to justify his action was that these were "midnight
appointments" that are forbidden under Article VII, Section 15
of the Constitution. However, the CSC ruled, and correctly so,
that the said prohibition applies only to presidential
appointments. In truth and in fact, there is no law that
prohibits local elective officials from making appointments
during the last days of his or her tenure. Petitioner certainly
did not raise the issue of fraud on the part of the outgoing
mayor who made the appointments. Neither did he allege
that the said appointments were tainted by irregularities or
anomalies that breached laws and regulations governing
appointments. His solitary reason for recalling these
appointments was that they were, to his personal belief,
"midnight appointments" which the outgoing mayor had no
authority to make.

HELD:
Prohibition under Section 15, Article VII does not apply to
appointments to fill a vacancy in the Supreme Court or to
other appointments to the Judiciary.
Had the framers intended to extend the prohibition
contained in Section 15, Article VII to the appointment of
Members of the Supreme Court, they could have explicitly
done so. They could not have ignored the meticulous ordering
of the provisions. They would have easily and surely written
the prohibition made explicit in Section 15, Article VII as
being equally applicable to the appointment of Members of the
Supreme Court in Article VIII itself, most likely in Section 4 (1),
Article VIII. That such specification was not done only
reveals that the prohibition against the President or Acting
President making appointments within two months before the
next presidential elections and up to the end of the Presidents
or Acting Presidents term does not refer to the Members of
the Supreme Court.

WHEREFORE, in view of all the foregoing, the instant petition


for review is DENIED and the Resolution of the Court of
Appeals in CA-G.R. SP No. 42896 affirming CSC Resolutions
Nos. 96-2828 and 96-7525 hereby AFFIRMED in toto.

Section 14, Section 15, and Section 16 are obviously of the


same character, in that they affect the power of the President
to appoint. The fact that Section 14 and Section 16 refer only to
appointments within the Executive Department renders
conclusive that Section 15 also applies only to the Executive
Department. This conclusion is consistent with the rule that
every part of the statute must be interpreted with reference to
the context, i.e. that every part must be considered together
with the other parts, and kept subservient to the general intent
of the whole enactment. It is absurd to assume that the framers
deliberately situated Section 15 between Section 14 and
Section 16, if they intended Section 15 to cover all kinds of
presidential appointments. If that was their intention in respect
of appointments to the Judiciary, the framers, if only to be clear,
would have easily and surely inserted a similar prohibition in
Article VIII, most likely within Section 4 (1)

DE CASTRO VS. JBC

Section 16: Power to Appoint

G.R. No. 191002, March 17, 2010

FACTS: The compulsory retirement of Chief Justice Reynato S.


Puno by May 17, 2010 occurs just days after the coming
presidential elections on May 10, 2010.
Under Section 4(1), in relation to Section 9, Article VIII, that
vacancy shall be filled within ninety days from the occurrence
thereof from a list of at least three nominees prepared by the
Judicial and Bar Council for every vacancy.
Also considering that Section 15, Article VII (Executive
Department) of the Constitution prohibits the President or
Acting President from making appointments within two months
immediately before the next presidential elections and up to
the end of his term, except temporary appointments to
executive positions when continued vacancies therein will
prejudice public service or endanger public safety.
The JBC, in its en banc meeting of January 18, 2010,
unanimously agreed to start the process of filling up the
position of Chief Justice.

government v springer
Sometime in the 1900s, the National Coal Company (NCC)
was created by the Philippine Congress. The law created it
(Act No. 2822) provides that: The voting power shall be
vested exclusively in a committee consisting of the GovernorGeneral, the President of the Senate, and the Speaker of the
House of Representatives.
In November 1926, the Governor-General (Leonard Wood)
issued E.O. No. 37 which divested the voting rights of the
Senate President and House Speaker in the NCC. The EO
emphasized that the voting right should be solely lodged in the
Governor-General who is the head of the government
(President at that time was considered the head of state but
does not manage government affairs). A copy of the said EO
was furnished to the Senate President and the House Speaker.

The OSG contends that the incumbent President may appoint


the next Chief Justice, because the prohibition under Section
15, Article VII of the Constitution does not apply to
appointments in the Supreme Court.

However, in December 1926, NCC held its elections and the


Senate President as well as the House Speaker,
notwithstanding EO No. 37 and the objection of the GovernorGeneral, still elected Milton Springer and four others as Board
of Directors of NCC. Thereafter, a quo warranto proceeding in
behalf of the government was filed against Springer et al
questioning the validity of their election into the Board of NCC.

ISSUE: Whether the incumbent President can appoint the


successor of Chief Justice Puno upon his retirement.

ISSUE: Whether or not the Senate President as well as the


House Speaker can validly elect the Board Members of NCC.

HELD: No. E.O. No 37 is valid. It is in accordance with the


doctrine of separation of powers. The Supreme Court
emphasized that the legislature creates the public office but it
has nothing to do with designating the persons to fill the office.
Appointing persons to a public office is essentially executive.
The NCC is a government owned and controlled corporation. It
was created by Congress. To extend the power of Congress
into allowing it, through the Senate President and the House
Speaker, to appoint members of the NCC is already an
invasion of executive powers. The Supreme Court however
notes that indeed there are exceptions to this rule where the
legislature may appoint persons to fill public office. Such
exception can be found in the appointment by the legislature of
persons to fill offices within the legislative branch this
exception is allowable because it does not weaken the
executive branch.
Bermudez vs. Torres G.R. No. 131429, August 4, 1999
Facts: Petitioner Oscar Bermudez, the First Assistant
Provincial Prosecutor of Tarlac and Officer-in-Charge of the
Office of Provincial Prosecutor, was a recommendee of then
Sec. of Justice Guingona for the position of Provincial
Prosecutor. Private respondent Atty. Conrado Quiaoit had the
support of then Representative Yap of the Second District of
Tarlac. Quiaoit was appointed by Pres. Ramos to the office.
Quiaoit took his oath and assumed office. Bermudez refused to
vacate the Office of the Provincial Prosecutor. Nonetheless,
Quiaoit, performed the duties and functions of the Office of
Provincial Prosecutor. Petitioner Bermudez challenged the
appointment of Quiaoit primarily on the ground that the
appointment lacks the recommendation of the Sec. Of Justice
prescribed under the Revised Administrative Code of 1987.
Section 9, Chap. II, Title III, Book IV of the Revised
Administrative Code provides that all provincial and city
prosecutors and their assistants shall be appointed by the
Pres. upon the recommendation of the Secretary.
Issue: Whether or not the absence of a recommendation of the
Secretary of Justice to the President can be held fatal to the
appointment of Quiaoit
Held: Section 9, Chap. II, Title III, Book IV of the Revised
Administrative Code ALL PROVINCIAL AND CITY
PROSECUTORS AND THERIASSISTANTS SHALL BE
APPOINTED BY THE PRESIDENT UPON THE
RECOMMENDATION OF THE SECRETARY
Appointment calls for discretion on the part of the appointing
authority. The power to appoint prosecutors is given to the
President. The Secretary of Justice is under the control of the
President. Hence the law must be read simply as allowing the
Secretary of Justice to advice the President.
Datu Michael Abas Kida v. Senate of the Philippines
FACTS

Several laws pertaining to ARMM were enacted


by Congress. RA 6734 is the organic act that established the
ARMM and scheduled the first regular elections for the ARMM
regional officials. RA 9054 amended the ARMM Charter and
reset the regular elections for the ARMM regional officials to
the second Monday of Sep 2001. RA 9140 further reset the
first regular elections to Nov 26, 2001. RA 9333 reset for the
third time the ARMM regional elections to the 2nd Monday of
Aug 2005 and on the same date every 3 years thereafter.
Pursuant to RA 9333, the next ARMM regional
elections should have been held on Aug 8, 2011. COMELEC
had begun preparations for these elections and had accepted

certificates of
elected. But
resetting the
coincide with
country.

candidacies for the various regional offices to be


on June 30, 2011, RA No. 10153 was enacted,
next ARMM regular elections to May 2013 to
the regular national and local elections of the

In these consolidated petitions filed directly with the


Supreme Court, the petitioners assailed the constitutionality
of RA No. 10153.
The petitions assailing RA 10153 maintain that it is
unconstitutional for its failure to comply with the three-reading
requirement of Section 26(2), Article VI of the Constitution. Also
cited as grounds are the alleged violations of the right of
suffrage of the people of ARMM, as well as the failure to
adhere to the "elective and representative" character of the
executive and legislative departments of the ARMM. Lastly, the
petitioners challenged the grant to the President of the
power to appoint OICs to undertake the functions of the
elective ARMM officials until the officials elected under the
May 2013 regular elections shall have assumed office.
Corrolarily, they also argue that the power of appointment
also gave the President the power of control over the
ARMM, in complete violation of Section 16, Article X of the
Constitution
ISSUES: (relating to the topic
1.
Does the 1987 Constitution mandate the synchronization
of elections [including the ARMM elections]?
2.
Does the passage of RA No. 10153 violate the threereadings-on-separate-days rule under Section 26(2), Article
VI of the 1987 Constitution?
3. Is the grant [to the President] of the power to appoint OICs
constitutional?
III. THE RULING
[The Supreme Court] DISMISSED the petitions and
UPHELD the constitutionality of RA No. 10153 in toto.]
1.
YES, the 1987 Constitution mandates the
synchronization of elections.
In this case, the ARMM elections, although called
regional elections, should be included among the elections to
be synchronized as it is a local election based on the wording
and structure of the Constitution.
Thus, it is clear from the foregoing that the 1987
Constitution mandates the synchronization of elections,
including the ARMM elections.
2.
NO, the passage of RA No. 10153 DOES NOT violate
the three-readings-on-separate-days requirement in
Section 26(2), Article VI of the 1987 Constitution.
In the present case, the records show that the
President wrote to the Speaker of the House of
Representatives to certify the necessity of the immediate
enactment of a law synchronizing the ARMM elections with the
national and local elections. Following our Tolentino ruling, the
Presidents certification exempted both the House and the
Senate from having to comply with the three separate readings
requirement.
3.
YES, the grant [to the President] of the power to
appoint OICs in the ARMM is constitutional

The Court identified the three options open to


Congress in order to resolve the problem on who should sit as
ARMM officials in the interim [in order to achieve
synchronization in the 2013 elections]: (1) allow the
[incumbent] elective officials in the ARMM to remain in office in
a hold over capacity until those elected in the synchronized
elections assume office; (2) hold special elections in the
ARMM, with the terms of those elected to expire when those
elected in the [2013] synchronized elections assume office; or
(3) authorize the President to appoint OICs, [their respective
terms to last also until those elected in the 2013 synchronized
elections assume office.]

The above considerations leave only Congress


chosen interim measure RA No. 10153 and the appointment
by the President of OICs to govern the ARMM during the presynchronization period pursuant to Sections 3, 4 and 5 of this
law as the only measure that Congress can make.

3.1.
1st option: Holdover is unconstitutional since it
would extend the terms of office of the incumbent ARMM
officials

This provision classifies into four groups the officers


that the President can appoint. These are:

We rule out the option since it violates Section 8,


Article X of the Constitution. This provision states:
Section 8. The term of office of elective local officials,
except barangay officials, which shall be determined by
law, shall be three years and no such official shall serve for
more than three consecutive terms.
Since elective ARMM officials are local officials, they
are covered and bound by the three-year term limit prescribed
by the Constitution; they cannot extend their term through a
holdover. xxx.
If it will be claimed that the holdover period is
effectively another term mandated by Congress, the net result
is for Congress to create a new term and to appoint the
occupant for the new term. This view like the extension of the
elective term is constitutionally infirm because Congress
cannot do indirectly what it cannot do directly, i.e., to act in a
way that would effectively extend the term of the incumbents.
Congress, in passing RA No. 10153, made it explicitly
clear that it had the intention of suppressing the holdover rule
that prevailed under RA No. 9054 by completely removing this
provision.
3.2.
2nd option: Calling special elections is
unconstitutional since COMELEC, on its own, has no
authority to order special elections.
The power to fix the date of elections is essentially
legislative in nature. [N]o elections may be held on any other
date for the positions of President, Vice President, Members of
Congress and local officials, except when so provided by
another Act of Congress, or upon orders of a body or officer to
whom Congress may have delegated either the power or the
authority to ascertain or fill in the details in the execution of that
power.
Congress has acted on the ARMM elections by
postponing the scheduled August 2011 elections and setting
another date May 13, 2011 for regional elections
synchronized with the presidential, congressional and other
local elections. By so doing, Congress itself has made a policy
decision in the exercise of its legislative wisdom that it shall not
call special elections as an adjustment measure in
synchronizing the ARMM elections with the other elections.
3.3.
3rd option: Grant to the President of the power to
appoint ARMM OICs in the interim is valid.

At the outset, the power to appoint is essentially


executive in nature, and the limitations on or qualifications to
the exercise of this power should be strictly construed; these
limitations or qualifications must be clearly stated in order to be
recognized. The appointing power is embodied in Section 16,
Article VII of the Constitution.

First, the heads of the executive departments;


ambassadors; other public ministers and consuls; officers of
the Armed Forces of the Philippines, from the rank of colonel or
naval captain; and other officers whose appointments are
vested in the President in this Constitution;
Second, all other officers of the government whose
appointments are not otherwise provided for by law;
Third, those whom the President may be authorized by
law to appoint; and
Fourth, officers lower in rank whose appointments the
Congress may by law vest in the President alone.
Since the Presidents authority to appoint OICs
emanates from RA No. 10153, it falls under the third group of
officials that the President can appoint pursuant to Section 16,
Article VII of the Constitution. Thus, the assailed
law facially rests on clear constitutional basis.
Kinds of Presidential Appointment
a. Submission to Commission on Appointments
aa. regular appointment
ab. ad interim appointments
Pimentel vs. Ermita
Facts: This is a petition to declare unconstitutional the
appointments issued by President Gloria Macapagal-Arroyo
through Executive Secretary Eduardo R. Ermita to Florencio B.
Abad, Avelino J. Cruz, Jr., Michael T. Defensor, Joseph H.
Durano, Raul M. Gonzalez, Alberto G. Romulo, Rene C. Villa,
and Arthur C. Yap as acting secretaries of their respective
departments.
On August 2004, Arroyo issued appointments to respondents
as acting secretaries of their respective departments.
Congress adjourned on 22 September 2004. On 23 September
2004, President Arroyo issued ad interim appointments to
respondents as secretaries of the departments to which they
were previously appointed in an acting capacity.
Issue: Is President Arroyo appointment of respondents as
acting secretaries without the consent of the Commission on
Appointments while Congress is in session, constitutional?
Held: Yes. The power to appoint is essentially executive in
nature, and the legislature may not interfere with the exercise
of this executive power except in those instances when the
Constitution expressly allows it to interfere. Limitations on the
executive power to appoint are construed strictly against the
legislature. The scope of the legislature\'92s interference in the
executive\'92s power to appoint is limited to the power to
prescribe the qualifications to an appointive office. Congress

cannot appoint a person to an office in the guise of prescribing


qualifications to that office. Neither may Congress impose on
the President the duty to appoint any particular person to an
office.

WHEREFORE, we DISMISS the present petition for certiorari


and prohibition.

However, even if the Commission on Appointments is


composed of members of Congress, the exercise of its powers
is executive and not legislative. The Commission on
Appointments does not legislate when it exercises its power to
give or withhold consent to presidential appointments

Sarmiento III vs. Mison

Petitioners contend that President Arroyo should not have


appointed respondents as acting secretaries because in case
of a vacancy in the Office of a Secretary, it is only an
Undersecretary who can be designated as Acting Secretary.
The essence of an appointment in an acting capacity is its
temporary nature. It is a stop-gap measure intended to fill an
office for a limited time until the appointment of a permanent
occupant to the office. In case of vacancy in an office occupied
by an alter ego of the President, such as the office of a
department secretary, the President must necessarily appoint
an alter ego of her choice as acting secretary before the
permanent appointee of her choice could assume office.
Congress, through a law, cannot impose on the President the
obligation to appoint automatically the undersecretary as her
temporary alter ego. An alter ego, whether temporary or
permanent, holds a position of great trust and confidence.
Congress, in the guise of prescribing qualifications to an office,
cannot impose on the President who her alter ego should be.
The office of a department secretary may become vacant while
Congress is in session. Since a department secretary is the
alter ego of the President, the acting appointee to the office
must necessarily have the President\'92s confidence. Thus, by
the very nature of the office of a department secretary, the
President must appoint in an acting capacity a person of her
choice even while Congress is in session. That person may or
may not be the permanent appointee, but practical reasons
may make it expedient that the acting appointee will also be
the permanent appointee.
Finally, petitioners claim that the issuance of appointments in
an acting capacity is susceptible to abuse. Petitioners fail to
consider that acting appointments cannot exceed one year as
expressly provided in Section 17(3), Chapter 5, Title I, Book III
of EO 292. The law has incorporated this safeguard to prevent
abuses, like the use of acting appointments as a way to
circumvent confirmation by the Commission on Appointments.
Ad-interim appointments must be distinguished from
appointments in an acting capacity. Both of them are effective
upon acceptance. But ad-interim appointments are extended
only during a recess of Congress, whereas acting
appointments may be extended any time there is a vacancy.
Moreover ad-interim appointments are submitted to the
Commission on Appointments for confirmation or rejection;
acting appointments are not submitted to the Commission on
Appointments. Acting appointments are a way of temporarily
filling important offices but, if abused, they can also be a way of
circumventing the need for confirmation by the Commission on
Appointments.
However, we find no abuse in the present case. The absence
of abuse is readily apparent from President Arroyo's issuance
of ad interim appointments to respondents immediately upon
the recess of Congress, way before the lapse of one year.

Scope of the power of the Commission on Apppointments

Facts: Petitioners seek to enjoin respondent Mison from


performing the functions of the Office of Commissioner of the
Bureau of Customs and respondent Carague as Secretary of
the Dept of Budget from disbursing payments for Misons
salaries and emoluments on the ground that Misons
appointment as Commissioner of the Bureau of Customs is
unconstitutional by reason of its not having been confirmed by
the Commission on Appointments (CA). On the other hand,
respondents maintain the constitutionality of Misons
appointment without the confirmation of the (CA). It is apparent
in Sec 16, Art. 7 of the Constitution that there are four groups
of officers whom the president shall appoint.
(1) the heads of the exec departments, ambassadors, other
public ministers and consuls, officers of the armed forces from
the rank of colonel or naval captain, and other officers whose
appointments are vested in him in the Constitution,
(2) all other officers of the Government whose appointments
are not otherwise provided for by law,
(3) those whom the President may be authorized by law to
appoint and
(4) officers lower in rank whose appointments the Congress
may by law vest in the President alone.
The 1st group is clearly appointed with the consent of the CA.
The 2nd, 3rd and 4th groups are the present bone of
contention.
Issue: Whether or not the 2nd, 3rd and 4th groups should be
appointed by the president with or without the consent/
confirmation of the CA
Held: The fundamental principle of Constitutional construction
is to give effect to the intent of the framers of the organic law
and the people adopting it. The Court will thus construe the
applicable constitutional provisions not in accordance with how
the executive or the legislative may want them construed, but
in accordance with what they say and provide. The 1935
Constitution requires confirmation by the CA of all presidential
appointments. This has resulted in horse-trading and similar
malpractices. Under the 1973 Constitution, the president has
the absolute power of appointment with hardly any check on
the legislature. Given these two extremes, the 1987
Constitution struck a middle-ground by requiring the consent
of the CA for the 1st group of appointments and leaving to the
President without such confirmation the appointments of the
other officers. The clear and expressed intent of the framers of
the 1987 Constitution is to exclude presidential appointments
from confirmation on the CA except appointments to offices
expressly mentioned in the first sentence of Sec. 16, Art VII.
Therefore, the confirmation on the appointment of
Commissioners of the Bureau of Customs by the CA is not
required.
The appointment of Mison without submitting his nomination
the CA is within the constitutional authority of the President.
WHEREFORE, the petition and petition in intervention should
be, as they are, hereby DISMISSED.
Bautista vs Salonga

FACTS:
The President appointed Mary Concepcion Bautista as the
Chairman of the Commission on Human Rights pursuant to the
second sentence in Section 16, Art. VII, without the
confirmation of the CoA because they are among the officers of
government "whom he (the President) may be authorized by
law to appoint." Section 2(c), Executive Order No. 163,
authorizes the President to appoint the Chairman and
Members of the Commission on Human Rights. CoA
disapproved Bautista's alleged ad interim appointment as
Chairperson of the CHR in view of her refusal to submit to the
jurisdiction of the Commission on Appointments.
ISSUES:
1. Whether or not Bautista's appointment is subject to CoA's
confirmation.
2. Whether or not Bautista's appointment is an ad interim
appointment.
RULING:
1. No. The position of Chairman of CHR is not among the
positions mentioned in the first sentence of Sec. 16 Art 7 of the
Constitution, which provides that the appointments which are
to be made with the confirmation of CoA. Rather, it is within the
authority of President, vested upon her by Constitution (2nd
sentence of Sec. 16 Art 7), that she appoint executive officials
without confirmation of CoA.

that sectoral representatives must first be confirmed by


the respondent Commissionbefore they could take their
oaths and/or assume office as members of the House of
Representatives. This opposition compelled Speaker Ramon
V. Mitra, Jr. to suspend the oath-taking of the four sectoral
representatives.
On May 10, 1988, petitioner Deles received an invitation dated
May 6, 1988 to attend a Commission on Appointments
Committee Meeting scheduled for May 12, 1988 for the
deliberation of her appointment as sectoral representative for
women. Petitioner sent a reply dated May 11, 1988 explaining
her position and questioning the jurisdiction of the Commission
on Appointments over the appointment of sectoral
representatives
Petitioner Quintos-Deles contends that her appointment as
Sectoral Representative for Women by the President pursuant,
does not require confirmation by the Commission on
Appointments to qualify her to take her seat in the House of
Representatives.
ISSUE:Whether or not the Constitution requires the
appointment of sectoral representatives to the House of
Representatives to be confirmed by the Commission on
Appointments.

WHEREFORE, the petition is GRANTED. Petitioner Bautista is


declared to be, as she is, the duly appointed Chairman of the
Commission on Human Rights and the lawful incumbent
thereof, entitled to all the benefits, privileges and emoluments
of said office. The temporary restraining order heretofore
issued by the Court against respondent Mallillin enjoining him
from dismissing or terminating personnel of the Commission on
Human Rights is made permanent.

HELD: Yes. The first group of people that may be appointed by


the president, as previously stated in the Sarmiento v. Mison
case, are the heads of the executive departments,
ambassadors, other public ministers and consuls, or officers of
the armed forces from the rank of colonel or naval captain, and
other officers whose appointments are vested in him in this
Constitution. Since the seats reserved for sectoral
representatives in paragraph 2, Section 5, Art. VI may be filled
by appointment by the President by express provision of
Section 7, Art. XVIII of the Constitution, it is indubitable that
sectoral representatives to the House of Representatives
are among the other officers whose appointments are
vested in the President in this Constitution, referred to in
the first sentence of Section 16, Article VII (or the first group of
people who may be appointed) whose appointments are
subject to confirmation by the Commission on Appointments.
Deles' appointment was made pursuant to Art. VII, Section 16,
p.2 which gives the President the power to make
appointments during the recess of the Congress, whether
voluntary or compulsory, but such appointments shall be
effective only until disapproval by the Commission on
Appointments or until the next adjournment of the Congress.
The records show that Deles appointment was made on April
6, 1988 or while Congress was in recess (March 26, 1988 to
April 17, 1988); hence, the reference to the said paragraph 2 of
Section 16, Art. VII in the appointment extended to her.

QUINTOS-DELES VS CA

PobreVs. Mendieta

On April 6, 1988, QUINTOS-DELES and three others were


appointed Sectoral Representatives by the President pursuant
to Article VII, Section 16, paragraph 2 and Article XVIII, Section
7 of the Constitution.

These consolidated petitions under Rules 45 and 65


[PETITION FOR CERTIORARI lang to eh] of the Rules of
Court were filed by HermogenesPobre to set aside the
decision dated August 5, 1992 and writ of prohibitory injunction
dated August 19, 1992 issued by Judge (now Court of Appeals
Justice) Corona Ibay-Somera, in Civil Case No. 92-60272
entitled, "Mariano A. Mendieta, petitioner vs. Hermogenes P.
Pobre, respondent," annulling the appointment extended by
P r e s i d e n t C o r a z o n C . A q u i n o t o t h e p e t i t i o n e r,
HermogenesPobre, as Commissioner/Chairman of the
Professional Regulation Commission (hereafter PRC for
brevity) and enjoining him from discharging the duties and
functions of that office.

The Commission on Appointments, by the actual exercise of its


constitutionally delimited power to review presidential
appointments, cannot create power to confirm appointments
that the Constitution has reserved to the President alone.
2. Under the Constitutional design, ad interim appointments do
not apply to appointments solely for the President to make. Ad
interim appointments, by their very nature under the 1987
Constitution, extend only to appointments where the review of
the Commission on Appointments is needed. That is why ad
interim appointments are to remain valid until disapproval by
the Commission on Appointments or until the next adjournment
of Congress; but appointments that are for the President solely
to make, that is, without the participation of the Commission on
Appointments, cannot be ad interim appointments.

On April 18, 1988, Quintos-Deles and other 3 sectoral


representatives were scheduled to take their oaths before
Speaker Ramon V. Mitra, Jr. at the Session Hall of Congress
after the Order of Business. However, Quintos-Deles and the
three other sectoral representatives- appointees were not able
to take their oaths and discharge their duties as members of
Congress due to the opposition of some congressmenmembers of the Commission on Appointments, who insisted

^^^ GRANTED YUNG PETITION, ANNULLED AND SET


ASIDE.
The controversy began on January 2, 1992, when the term of
office of Honorable Julio B. Francia as PRC Commissioner/
Chairman expired. At that time, Mariano A. Mendieta was the
senior Associate Commissioner and Hermogenes P. Pobre
was the second Associate Commissioner of the PRC.
On January 6, 1992, Executive Secretary Franklin M. Drilon
sought the opinion of Acting Secretary of Justice Silvestre H.
Bello, III on whether the President's power to appoint the
Commissioner of the Professional Regulation Commission is
restricted by Section 2 of P.D. No. 223, as amended, which
provides:
Sec. 2. Composition. The Commission
shall be headed by one fulltime
Commissioner and two fulltime Associate
Commissioners, all to be appointed by the
President for a term of nine (9) years without
reappointment to start from the time they
assume the office, except the first two
Associate Commissioners who shall be
appointed, one for six (6) years and the other
for three (3) years, and thereafter, any
vacancy in the Commission shall be filled
for the unexpired term only with the most
senior of the Associate Commissioner
succeeding the Commissioner at the
expiration of his term, resignation or
removal. No person shall be appointed
chairman or member of the Commission
unless he is at lease forty (40) years of age,
familiar with the principles and methods of
professional regulation and/or licensing and
has at least five (5) years of executive or
managerial experience.
On February 15, 1992, President Corazon C. Aquino appointed
the Pobre, then an Associate Commissioner, as the PRC
Commissioner/ Chairman. He took his oath of office on
February 17, 1992.
Even before Commissioner Pobre's appointment, the private
respondent, Mariano A. Mendieta, as the Senior Associate
Commissioner, filed a petition for declaratory relief against
Commissioner Pobre, Executive Secretary Drilon, and Acting
Secretary of Justice Eduardo Montenegro, praying that they be
enjoined from appointing, or recommending the appointment of
Associate Commissioner Pobre as Chairman of the PRC
because under Section 2 of P.D. No. 223, he (Mendieta), as
the senior Associate Commissioner, was legally entitled to
succeed Francia as Chairman of the PRC. His prayer for a
restraining order was set for hearing on February 19, 1992 at
2:30 o'clock in the afternoon.
Pobre opposed the issuance of a restraining order because
President Aquino had already appointed him PRC Chairman
and he had, in fact, already taken his oath of office on February
17, 1992. Judge Somera denied the prayer for a restraining
order as well as the petition for declaratory relief for being moot
and academic.
[IN SHORT, BITTER NA NAMANITONG SI MENDIETA kay
POBRE, kasisiyadawang senior Associate Commissioner
at bakithindi raw siyayungpumalit kay francia]
Issue: Whether the President may appoint as Commissioner/
Chairman of the PRC another Associate Commissioner or any
person other than the Senior Associate Commissioner
Held:Yes. The petition raises an issue regarding the proper
construction of the provision in Section 2 of P.D. No. 223 that:
. . . any vacancy in the Commission shall be filled for the
unexpired term only with the most Senior of the Associate
Commissioners succeeding the Commissioner at the expiration

of his term, resignation or removal, whereby the legality of


Pobres appointment as PRC Chairman may be determined.
The succession clause operates only when there is an
unexpired term of the Chairman/Commissioner to be served.
Otherwise, if the Chairmans term had expired or been fully
served, the vacancy must be filled by appointment of a new
chairman by the President.
The appointment of petitioner Hermogenes P. Pobre as
Commissioner/Chairman of the Professional Regulation
Commission is declared lawful and in order.
RUFINO vs. ENDRIGA
Facts:

Two consolidated petitions for review on certiorari


under rule 45 of the 1997 rules of civil procedure.
First case, GR No. 139554, represented by the Solicitor
General, collectively known as the RUFINO group seeks to set
aside the Decision of the Court of Appeals and the Resolution
denying the motion for the reconsideration.
CAs decision (to be set aside)
1.
Declaring petitioners, ENDRIGA group to have a
clear right to their respective offices elected by the
CCP board up to expiration of 4-year term
2.
Ousting respondents, Rufino group, except
respondent Tantoco, from their respective offices and
excluding them therefrom
3.
Dismissing case against Tantoco
Second case, GR No. 139565, the Endriga group, seeks to
assail the Resolution issued by the Court of Appeals in same
case insofar as it denied their Motion for Immediate Execution
of the Decision.
History of PD 15:
Marcos In 1966, Marcos created EO 30 Creation of the
Cultural Center of the Philippines as a trust governed by a
Board of Trustees of 7 members to preserve and promote
Philippine culture. The original founding trustees were all
pointed by Pres. Marcos
1972, after declaration of Martial Law Pres. Marcos issued
PD 15 which converted CCP into a non-municipal public
corporation free from the pressure or influence of politics and
increased 7 members to 9. EO 1058 issued in 1985 increased
9 to 11.
Aquino - 1986, after People Power Resolution, Aquino asked
incumbent CCP trustees for courtesy resignation and
appointed new trustees to the Board.
Ramos started Endriga group
Estrada appointed 7 new trustees to CCP board with term of
4 years to replace the Endriga group, except for Tantoco. Thus
Rufino group took respective oaths of office and assumed
performance of their duties in 1999.
Endriga vs. Estrada

Endriga group files Petition for Quo Warranto


questioning Pres. Estradas appointment of 7 new
member of CCP board.

Alleged that under Sec. 6 (b) of PD 15 vacancies


of the CCP Board shall be filled by election by
majority vote of trustees held at next regular
meeting only when the board is entirely vacant

may the President of the Phil. fill such vacancies,


acting in consultation with ranking officers of CCP. In
the case at bar, only one seat was vacant due to Ma
Osas expiration term.
4-year term: Endriga group maintained that under
CCP Charter, the trustees fixed 4-year term can only
be terminated by reason of resignation, incapacity,
death, or other cause thus Pres. action was
unnecessary because 10 incumbent trustees had the
statutory power to fill up by election any vacancy of
the board.
Sec. 3 of PD 15 Endriga refuses to accept CCP is
under the supervision and control of the President as
Sec. 3 states shall enjoy autonomy of policy and
operation
CAs decision: declared Endriga group lawfully
entitled to hold office as CCP trustees and ousting
Rufino group from board.
Sec. 6 (b) of PD 15 is free from ambiquity.
Nor may the Presidents constitutional statutory
power of supervision and control over
government corporations restrict or modify
application of CCP Charter.
Rufinos Motion for Recon: asserted, by Section 16
of Article 7 of the 1987 Consti, that the law could
only delegate to the CCP board the power to appoint
officers lower in rank than trustees of Board which is
in conflict with Sec. 6 of PD 15 CCP trustees has
the authority to appoint or elect fellow officers of
equal rank not of lower
CA: denied Rufinos Motion and also denied
Endrigas motion for immediate execution of
decision.
Hence the consolidated petitions.

MAIN ISSUE OF 1st case: Whether or not Section 6 paragraph


B of PD 15 in which gives authority to CCP trustees to elect or
appoint fellow officers of equal rank, is unconstitutional (as it is
against Sec.16 of Article 7, that only the President has the
appointing powers of appointing heads of executive
departments)
Important Issues related to lesson:
1.
Invalid delegation of the Presidents
appointing power under the Consti
2.
Deprives Presidents constitutional power of
control and supervision over CCP
MAIN ISSUE OF 2nd case: whether or not a writ of quo
warranto involving public office such as CCP should be
declared a self-executing judgment and deemed immediately
executor under Rule 39, Section 4 of the rules of court.

Appointment of Heads of Departments, Agencies,


Commissions, and Boards:
Appointing powers belong to President, with:
Congress - share such authority as to
o
appointing inferior or lower in rank than
those vested by law
BatasangPambansa can also appoint
o
inferior officers
Power to appoint to heads is a matter of
legislative grace.
Presidents power is self-executing vested by
Consti thus not subject to legislative limitations
or conditions. Others such power are the
Supreme Court en banc and Consti Com
Sec. 8 PD 15 Chairperson of CCP Board is Head of CCP:
Chairperson, with confirmation from the Board,
has the power to appoint all officers, staff and
personnel of the Center
The CCP may elect membership in Govt
Service Insurance System (GSIS) those elected
will have same rights and privileges and
obligations as govt service
Exempted from coverage of Civil Service Law
and Rules
Sec. 3 Duties of Chairperson:
Appoint, remove, discipline all officers and
personnel
Perform duties until Board, by majority vote,
shall elect another Chairperson

Section 6 (b) and (c) of PD 15:


Board of trustees has the power and authority of the
corporation. Vacancies shall be filled by election of majority
vote by the trustees. Only can the President exercise his/her
power to appoint if the board becomes entirely vacant.
Intent to insulate the CCP from political influence and
pressure, specifically from the Pres. Makes CCP a selfperpetuating entity, virtually outside the control of the Pres.
SUCH CANNOT EXISTS UNDER CONSTI. Sec. 3s provision
of Board initiating and formulating plocies and activities are still
subject to Pres. power of control.
Section 16 of Article 7 of Consti:
source of presidents power to appoint
-

gives President legislature authority to delegate


power to appoint
However, Congress may, by law, vest upon the
appointment of other officers lower in rank (as
contended in Rufinos MOC)
President appoints 5 groups of officers:
1.
Heads of Executive department (with
consent from Commission of Appointments)
2.
Those whom President may be authorized
by law to appoint (and 3. without consent of
Comm of App)
3.
Officers of the Government (if the law is
silent or head appointing declared unconsti)
4.
Lower-ranked officers whose appointments
Congress may, by the law, vest in the heads
of departments, agencies, commissions or
boards.
5.
Appointments vested by Consti in Supreme
Court and Constitutional Commission

Thus, under COnsti, CCP head is Chairperson and


has power to appoint lower ranked officers
Under PD 15, CCP is a public corporation governed
by the Board thus not an agency

Sec. 6 (b) (c) vs. Sec. 16 of Article 7:


HOLDING: Sec. 6 (b) and (c) of PD 15 are unconstitutional.
While Sec. 6 empowers remaining trustees to fill
vacancies of Board, allowing them to elect fellow
trustees... Sec. 16 allows only the heads of
departments, agencies, commissions or boards
to appoint only officers lower in rank
Sec. 6 talks about independent appointing
powers which conflicts with the Presidents
power to appoint the two system of appointing
powers are recurring anomalies and

controversies in appointment every time new


President assumed office.
What is the CCP?
CCP is under the Executive branch, as stated in
Revised Admin Code of 1987 any agency, not
placed or order creating them under specific
department falls under Office of Pres.
President controls CCP
Sec. 17 of Art. 7 CCP perform executive and
not legislative, judicial or quasi-judicial functions.
Presidents Power to Control:
Applies to acts or decision of all officers in the
Executive Branch
Power to control the power to revise or
reverse acts or decisions of a subordinate officer
involving exercise of discretion
Supervision and Control includes authority to
act directly whenever a specific function is
entrusted by law or regulation of subordinate
Executive branch is unitary thus only the
Presindet had executive power exercising
control over entire Executive Branch.
Legislature cant validly enact law outside
control of Presindent.
Limitations:
Doesnt extend to qyaus-judicial bodies,
o
since proceedings and decisions are judicial
in nature and subject to judicial review, only
admin power of Pres.
Local government units only general
o
supervision
HOLDING:
Wherefore, we grant the petition of the 1st case
wherein we find Sec. 6 (b) and (c) of PD 15
UNCONSTITUTIONAL insofar as it authorizes remaining
trustees to fill by election vacancies of the Board. And we find it
unnecessary to rule 2nd case.
Main Point of the Case:
The power to appoint is the prerogative of the President except
in those instances when the Constitution provides otherwise.
Under Section 16, there is a fourth group of lower-ranked
officers whose appointments, Congress may by law vest in the
heads of departments, agencies commissions or boards.
Congress has the discretion to grant, or withhold from, the
heads of the department has the power to appoint lowerranked officials. The 1987 Constitution only allows heads of
departments, agencies, commissions or boards to appoint
ONLY.
Congress Requiring Confirmation by the Commission on
Appointments
CALDERON vs. CARALE
Facts:
In 1989, RA 6715 was passed. This law amended PD 442 or
the Labor Code. RA 6715 provides that the Chairman, the
Division Presiding Commissioners and other Commissioners
[of the NLRC] shall all be appointed by the President, subject
to confirmation by the CoA. Appointments to any vacancy shall
come from the nominees of the sector which nominated the
predecessor. Pursuant to the law, Cory assigned Carale et al
as the Chairman and the Commissioners respectively of the
NLRC, the appointment was not submitted to the CoA for its

confirmation. Calderon questioned the appointment saying that


w/o the confirmation by the CoA, such an appointment is in
violation of RA 6715. Calderon asserted that RA 6715 is not an
encroachment on the appointing power of the executive
contained in Sec16, Art. 7, of the Constitution, as Congress
may, by law, require confirmation by the Commission on
Appointments of other officers appointed by the President
additional to those mentioned in the first sentence of Sec 16 of
Article 7 of the Constitution.
Issue:
Whether or not Congress may, by law, require confirmation by
the CoA of appointments extended by the President to
government officers additional to those expressly mentioned in
the first sentence of Sec. 16, Art. 7 of the Constitution whose
appointments require confirmation by the CoA.
Ruling:
The SC agreed with the Sol-Gen, confirmation by the CoA is
required exclusively for the heads of executive departments,
ambassadors, public ministers, consuls, officers of the armed
forces from the rank of colonel or naval captain, and other
officers whose appointments are vested in the President by the
Constitution, such as the members of the various
Constitutional Commissions. With respect to the other officers
whose appointments are not otherwise provided for by the law
and to those whom the President may be authorized by law to
appoint, no confirmation by the Commission on Appointments
is required.
Jurisprudence established the following in interpreting Sec 16,
Art 7 of the Constitution
1.
Confirmation by the Commission on Appointments is
required only for presidential appointees mentioned
in the first sentence of Section 16, Article VII,
including, those officers whose appointments are
expressly vested by the Constitution itself in the
president (like sectoral representatives to Congress
and members of the constitutional commissions of
Audit, Civil Service and Election).
2.

Confirmation is not required when the President


appoints other government officers whose
appointments are not otherwise provided for by law
or those officers whom he may be authorized by law
to appoint (like the Chairman and Members of the
Commission on Human Rights).

U-sing vs. NLRC


Facts:
This is a special civic action that seeks the reversal of the
resolution made by the NLRC dismissing the appeal of the
petitioner (Cecilia Naya vs. U-sing Button & Buckle Industry
represented by Mr. Sy Ban)
Antecedent of the case:
In June, 1960, Fortunato Naya was employed as a
maintenance worker at U-Sing Button and Buckle Industry
located at 158 4th Street, 7th Avenue, Grace Park, Caloocan
City. The establishment, as its name suggests, is engaged in
the manufacture and sale of buttons and buckles. On May 30,
1986, Naya stopped working on account of illness and died
shortly thereafter. His widow, Cecilia Naya, filed in the Manila
Arbitration Branch of the Department of Labor & Employment
against U-Sing Button and Buckle Industry and its proprietor
and general manager, Sy Ban, a claim for separation pay and
incentive leave pay due her husband. The U-sing, in order to
escape liability, presented evidence of Fortunato Naya's
alleged indebtedness to them in the amount of 116,500.00.

They further claimed that they gave him P4, 247.00 during his
confinement at the Lung Center of the Philippines and donated
123,500.00 to his family when he died.
Cecilia contended that her signature were forged being noted
by the labor arbiter, Daisy Cauton-Barcelona, the difference
and dissimilarities in the signatures on the receipts presented
by the U-Sing. A decision was rendered on September 21,
1989 dismissing for lack of factual basis the claim for
underpayment but ordered the respondents to pay the
complainant separation benefits owing to her late husband,
computed from June 1960 to May 1986 at the rate of one-half
month pay for every year of service.
U-sing appealed contending that the NLRC has no jurisdiction
over the case, because "the appointments of the
commissioners have not been confirmed by the Commission
on Appointments;"2. the failure of the petitioners to file a surety
bond is not a valid ground for the dismissal of the appeal;
and3. the NLRC erred in not rendering judgment in favor of the
petitioners and against the private respondent, Cecilia Naya.
Issue:
Whether or not the NLRC commissioner was among the
officers that requires confirmation from the Commission on
Appointments
Ruling:
It does not, non-confirmation by the Commission on
Appointments of the new NLRC Commissioners who were
appointed under Republic Act 6715 did not make their
appointment null and void. In Calderoti vs. Carale, 208 SCRA
254, The SC held that the Chairman and members of the
National Labor Relations Commission are not among the
officers mentioned in Section 16, Article VII of the 1987
Constitution whose appointments require confirmation by the
Commission on Appointments. Therefore, their acts are valid.
In any case, the petitioners raised this issue only in their
present petition, after their motion for reconsideration was
denied by the Commission. They are estopped from
repudiating the jurisdiction of the NLRC which they had already
recognized.
Tarrosa v. Singson
FACTS:
Gabriel C. Singson was appointed Governor of the Bangko
Sentral by President Fidel V. Ramos in 1993.
Jesus Armando Tarrosa, as a "taxpayer", filed a petition for
prohibition questioning the appointment of Singson for not
having been confirmed by the Commission on Appointments as
required by the provisions of Section 6 of R.A. No. 7653, which
established the Bangko Sentral as the Central Monetary
Authority of the Philippines.
The Secretary of Budget and Management was impleaded for
disbursing public funds in payment of the salaries and
emoluments of respondent Singson.
In their comment, respondents claim that Congress exceeded
its legislative powers in requiring the confirmation by the CA of
the appointment of the Governor of the Bangko Sentral. They
contend that an appointment to the said position is not among
the appointments which have to be confirmed by the CA, citing
Section 16 of Article VI of the Constitution.

ISSUE:
Whether or not the Governor of the BSP is subject to COAs
confirmation.
RULING:
PETITION DENIED.
Appointment to the position of the Governor of the BSP is not
one of those that need confirmation by the Commission on
Appointments. Congress cannot expand the confirmation
powers of the Commission on Appointments and require
confirmation of appointment of other government officials not
expressly mentioned in the first sentence of Section 16 of
Article VII of the Constitution.
Manalo v Sistoza
FACTS:
Republic Act No. 6975 was passed in 1990. This law created
the Department of Interior and Local Government.
Sections 26 and 31 thereof, also provided on the manner as to
how officers of the Philippine National Police are to be
appointed. It was provided that the PNP Chief as well as
certain police officers including Directors and Chief
Superintendents, after being appointed by the President, must
be confirmed by the Commission on Appointments before said
officers can take their office.
In 1992, Pres. Corazon Aquino appointed Pedro Sistoza et. al
as Directors and Chief Superintendents within the PNP.
Said appointments were not confirmed by the Commission on
Appointments.
Thus, Jesulito Manalo questioned the validity of the
appointments made. He contends that without the confirmation
by the Commission, Sistoza et. al are acting without
jurisdiction, their appointment being contrary to the provisions
of R.A. 6975.
He then went to the Supreme Court asking the court to carry
out the provisions of the said law. Manalo also insists that the
law is a valid law, as it enjoys the presumption of
constitutionality, and hence, it must be carried out by the
courts.
ISSUES:
1. Whether or not Sections 26 and 31 of RA 6975 are valid
2. Whether or not PNP officers are similar to the AFP, whose
positions need CA confirmation
3. Whether or not Salvador Enriquez III acted with grave abuse
of discretion
RULING:
PETITION DISMISSED.
1. Partly. Sections 26 and 31 go against the Constitution
because according to Sec. 16, Art. 7, the PNP officers do not
fall under the first group which requires the confirmation of the
CA. However, the remainder of RA 6975 remains valid
because assailed provisions are severable from the main
statute.
2. No. The constitution expressly provides for their distinction
according to Sections 24 and 6 of Art. 16. Also, RA 6975
provides that no element of the police force shall be military
nor shall any position thereof be occupied by active members
of the AFP.
3. No. The disbursements and emoluments disbursed for the
respondents are valid.

Soriano v. Lista
FACTS:
Eight officers of the Philippine Coast Guard (PCG) were
promoted by the President to Vice Admiral, Rear Admiral,
Commodore, Naval Captain, and they assumed office without
confirmation by the Commission on Appointments (COA).
Petitioner, as a taxpayer, filed a petition with the Supreme
Court questioning the constitutionality of their assumption of
office, which requires confirmation of the COA.
ISSUES:
1. Whether or not the petitioner has locus standi
2. Whether or not appointments of PCG officers, need CA
confirmation
RULING:
PETITION DISMISSED.
1. Petitioner has no locus standi. A party bringing a suit
challenging the constitutionality of an act must show not only
that the law or act is invalid, but also that he has sustained, or
is in immediate or imminent danger of sustaining some direct
injury as a result of its enforcement and not merely that he
suffers thereby in some indefinite way.
The instant petition cannot even be classified as a taxpayers
suit because petitioner has no interest as such and this case
does not involve the exercise by Congress of its taxing power.
2. Now that the PCG is under the DOTC and no longer part of
the Philippine Navy or the Armed Forces of the Philippines, the
promotions and appointments of respondent officers of the
PCG, or any PCG officer from the rank of captain and higher
for that matter, do not require confirmation by the CA.
Section 16, Article VII of the 1987 Constitution provides:
Section 16. The President shall nominate and, with the consent
of the Commission on Appointments, appoint the heads of the
executive departments, ambassadors, other public ministers
and consuls, or officers of the armed forces from the rank of
colonel or naval captain, and other officers whose
appointments are vested in him in this Constitution. He shall
also appoint all other officers of the Government whose
appointments are not otherwise provided for by law, and those
whom he may be authorized by law to appoint. The Congress
may, by law, vest the appointment of other officers lower in
rank in the President alone, in the courts, or in the heads of
departments, agencies, commissions, or boards.
The President shall have the power to make appointments
during the recess of the Congress, whether voluntary or
compulsory, but such appointments shall be effective only until
disapproval by the Commission on Appointments or until the
next adjournment of the Congress.
It is clear from the foregoing provision of the Constitution that
only appointed officers from the rank of colonel or naval
captain in the armed forces require confirmation by the CA.
The rule is that the plain, clear and unambiguous language of
the Constitution should be construed as such and should not
be given a construction that changes its meaning.
Section 17: Power of Control
LACSON-MAGALLANES V. PAO

Facts:
Jose Magallanes [co-owner of Lacson-Magallanes Co (LMC)]
was a permittee and actual occupant of a 1,103-hectare
pasture land in Davao. He ceded his rights of the land to LMC,
which upon official release, became an agricultural land.
Jose Pao [a farmer] and 19 othersapplie d for the purchase
just 90 hectares of the now agricultural land. LMC, however,
filed for sales application of the entire land.Pao protested that
they are actual occupants of the 90-ha portion.
The Director of Lands (DoL) gave due course to LMCs sales
application and dismissed Paos claims. Pao appealed to the
Secretary of Agriculture and Natural Resources (SoA&NR) but
was likewise dismissed. The case was elevated to the
President.
The Executive Secretary, by the authority of the President,
reversed the decision of the DoL and the SoA&NR stating that
it would be for the public interest that appellants, who are
mostly landless farmers who depend on the land for their
existence, be allocated that portion on which they have made
improvements.
LMC protested to the Court of First Instance and alleged that:
(1)
the decision of the Secretary of Agriculture and
Natural Resources has full force and effect;
(2)
the decision of the Executive Secretary is contrary to
law and of no legal force and effect.
Upon dismissal of the case, the LMC appealed to SC.
Issue related to Section 17:
1.
Ruling:

Can the Power of Control be delegated by the


President to the Executive Secretary?

Yes, the President can delegate the power of control to the


Executive Secretary except for cases where he is required by
the Constitution to act in person. The President is not
expected to perform in person all the multifarious executive
and administrative functions. Thus, the decision of the
Executive Secretary has full force and effect.
MACEDA V. MACARAIG
Facts:
The National Power Corporation (NPC) was created to
generate hydraulic power from all water sources in the
Philippines. Its main source of funds is through bonds, and
such bonds shall be exempted from taxes.
A lot of laws related to the NPC was enacted thereafter but
retained the exemption of the payment of the debt of NPC.
Then came PD 1931 which reiterated the repeal of all tax
exemptions of government-owned and controlled corporations,
but gave the power to the President and the Minister of
Finance to restore such exemptions taking into account the
following:
(1)
the effect on the relative price levels;
(2)
the relative contribution of the corporation to the
revenue generation effort;
(3)
the nature of the activity in which the corporation is
engaged in; or
(4)
In general, the greater national interest to be served.

Issue related to Section 17


1.

Is the NPC exempted from tax insofar as the


President has the power of control to exempt it from
such tax?

Ruling
Yes, the President has the power exempt NPC from tax for the
greater national interest that NPC will serve. Further rules
withdrawing the exemption bestowed upon the corporation
through the power vested upon the President cannot declare
such exemption invalid.
Roque v. Director of Lands (1976)
Facts:
In settling the dispute on the ownership of Lot No. 4507
between Roque who allegedly occupied the portion since 1937
and Facun who filed a homestead application in 1935 and
submitted final proof in 1939, the Department of Agriculture
and Natural Resources decided in favor of Roque. However,
upon re investigation it is found out that Roque submitted his
sales application for the disputed portion in 1948, only during
the course of the investigation of his protest. It was also
verified during the re investigation that Roque entered upon the
disputed portion in 1951. So the President, through respondent
Assistant Executive Secretary awarded the land in favor of the
Facun.
The Roque prayed that the order of the respondent Honorable
Director of Lands and the decision of the respondent
Honorable Assistant Executive Secretary, be set aside on the
alleged ground that the said order of the Director of Lands was
issued with grave abuse of discretion, consisting of unqualified
reliance and the biased report and recommendation. He also
asserted that the decision of the Honorable Executive
Secretary exceeded his jurisdiction and committed grave
abuse of discretion disregarding the sales award of the land in
question in favor of Facun having already paid for the price of
the same, and praying further that the decision of the
Honorable Secretary of Agriculture and Natural Resources
(which is in his favor) be sustained. Respondent Jose Facun,
through Atty. Cipriano A. Tan, filed an answer to the petition
denying specifically the allegation of abuse of discretion,
arbitrariness and excess of jurisdiction of the Honorable
Director of Lands and Assistant Executive Secretary with the
contention that it is perfectly valid.
Issue: W/N Assistant Executive Secretary lacks the power to
overrule the decision of Department of Agriculture and Natural
Resources
Ruling:
No, to contend that the Office of the President, through
respondent Assistant Executive Secretary, lacks the power to
overrule the Department of Agriculture and Natural Resources
is equivalent to questioning Art VIII Sec 17 of the 1987
Philippine Constitution. The President has control of all the
executive departments, bureaus or offices and that "The power
of control under this provision implies the right of the President
to interfere in the exercise of such discretion as may be vested
by law in the officers of the executive departments, bureaus, or
offices of the national government, as well as to act in lieu of
such officers." Clearly then, there is nothing to prevent the
President to disapprove the act of a department head.
Assistant Executive Secretary of the President is correct for
sustaining the award by the Director of Lands of a homestead

application and thus overruling the Secretary of Agriculture and


Natural Resources, because it is in conformity with the policy
of the law. Petitioner, himself a previous beneficiary of the
statute, would seek to add to his holding by a sales application.
The prevailing party, private respondent Jose Facun, on the
other hand, had applied for the disputed lot as a homesteader
as far back as 1935, and had submitted his final proof in 1939.
WHEREFORE, the appealed decision is affirmed. This
decision is immediately executory. Costs against petitioner
Ireneo Roque.
Ang-Angco v. Castillo (1963)
Facts: (medyo useless yung facts pero kase ang hirap
imanage nung flow ng case kung wala yung dates at batuhan
ng information; mahabang version)
On October 8, 1956, the Pepsi-Cola Far East Trade
Development Co., Inc. wrote a letter to the Secretary of
Commerce and Industry requesting for special permit to
withdraw certain commodities from the customs house not
covered by Central Bank release certificate which were
imported without any dollar allocation or remittance of foreign
exchange. On the same date, the company addressed an
identical request to the Secretary of Finance who was also the
Chairman of the Monetary Board of the Central Bank.
Senator Pedro Sabido, in behalf of the company, likewise wrote
said official urging that authority be given to withdraw the
commodities. Not content with this step, he also wrote to Dr.
Andres Castillo, Acting Governor of the Central Bank, urging,
the same matter.
Then Secretary Hernandez wrote another letter to Dr. Castillo
stating, "Senator Sabido is taking this to you personally. Unless
we have legal objection, I would like to authorize the
withdrawal of the concentrates upon payment of all charges in
pesos. Please expedite action."
Almost at the same time, the Import-Export Committee of the
Central Bank, thru Mr. Gregorio Licaros, submitted to the
Monetary Board a memorandum on the joint petition of the
company and Sabido Law Office for authority to withdraw the
concentrates from the customs house stating therein that it
sees no objection to the proposal.
The Monetary Board, however, failed to take up the matter in
its meeting of October 12, 1956 for the reason that the
transaction did not involve any dollar allocation or foreign
exchange.
Having failed to secure the necessary authority from the
Central Bank, on October 13, 1956, the counsel of the PepsiCola Far East Trade Development Co., Inc., approached
Collector of Customs Isidro Ang-Angco in an attempt to
secure from him the immediate release of the
concentrates, but this official seeing perhaps that the
importation did not carry any release certificate from the
Central Bank advised the counsel to try to secure the
necessary release certificate from the No-Dollar Import
Office that had jurisdiction over the case.
In the morning of the same day, Mr. Aquiles J. Lopez, of said
Office, wrote a letter addressed to the Collector of Customs
stating, among other things, that his office had no objection to
the release of the concentrates but that it could not take
action on the request as "the same is not within the
jurisdiction of the No-Dollar Import Office within the
contemplation of R.A. No. 1410."

This letter was shown to Collector of Customs Ang-Angco who


upon perusing it still hesitated to grant the release and
suggested that the letter be amended in order to remove
the ambiguity appearing therein, but Mr. Lopez refused to
amend the letter stating that the same was neither a permit
nor a release.

direct action and dispose of the administrative case in


question inasmuch as the provisions of law that would
seem to vest final authority in subordinate officers of the
executive branch of the government over administrative
matters falling under their jurisdiction cannot divest the
President of his power of control nor diminish the same.

Ang-Angco, while still in doubt as to the propriety of the


action suggested, finally authorized the release of the
concentrates upon payment of the corresponding duties,
customs charges, fees and taxes.

Hence, after exhausting all the administrative remedies


available to him to secure his reinstatement to the office from
which he was removed without any valid cause or in violation
of his right to due process of law, Collector Ang-Angco filed
before this Court the present petition for certiorari, prohibition
and mandamus with a petition for the issuance of a preliminary
mandatory injunction. The Court gave due course to the
petition, but denied the request for injunction.

When Commissioner of Customs Manuel P. Manahan learned


of the release of the concentrates in question he immediately
ordered their seizure but only a negligible portion thereof
remained in the warehouse. Whereupon, he filed an
administrative complaint against Collector of Customs
Ang-Angco charging him with having committed a grave
neglect of duty and observed a conduct prejudicial to the
best interest of the customs service.
On the strength of this complaint President Ramon Magsaysay
constituted an investigating committee to investigate AngAngco and Mr. Aquiles J. Lopez, who was also charged in a
separate complaint with serious misconduct in office or conduct
prejudicial to the best interest of the State.
As a result, Collector Ang-Angco was suspended from
office in the latter part of December, 1956. The committee in
its report is recommending that a suspension of 15 days,
without pay, be imposed upon Ang-Angco chargeable against
the period of his suspension.
On April 1, 1957, Collector Ang-Angco was reinstated to his
office by Secretary Hernandez, but the decision on the
administrative case against him remained pending until the
death of President Magsaysay. After around three years from
the termination of the investigation during which period AngAngco had been discharging the duties of his office, Executive
Secretary Natalio P. Castillo, by authority of the President,
rendered a decision on the case on February 12, 1960
finding Ang-Angco "guilty of conduct prejudicial to the
best interest of the service", and considering him resigned
effective from the date of notice, with prejudice to
reinstatement in the Bureau of Customs.
Ang-Angco wrote a letter to President Carlos P. Garcia calling
attention to the fact that the action taken by Secretary
Castillo in removing him from office had the effect of
depriving him of his statutory right to have his case
originally decided by the Commissioner of Civil Service,
as well as of his right of appeal to the Civil Service Board
of Appeals, whose decision under Republic Act No. 2260 is
final, besides the fact that such decision is in violation of
the guaranty vouchsafed by the Constitution to officers or
employees in the civil service against removal or
suspension except for cause in the manner provided by
law.
In a letter dated February 16, 1960, Secretary Castillo, also by
authority of the President, denied the request for
reconsideration. Not satisfied with this resolution, Collector
Ang-Angco sent a memorandum to President Garcia reiterating
once more the same grounds on which he predicated his
request for reconsideration. Again Secretary Castillo, also by
authority of the President, in letter dated July 1, 1960, denied
the appeal. In this instance, Secretary Castillo asserted that
the President virtue of his power of control over all
executive departments, bureaus and offices, can take

Facts (shortcut version hehehe):


Pepsi-Cola Far East Trade Development Co., Inc. wrote a
letter to the Secretary of Commerce and Industry requesting
for special permit to withdraw certain commodities from the
customs house not covered by Central Bank release
certificate. However after the long process of informing and
asking for permissions from the different offices, that pointed at
each other for actions on the aforementioned letter, it was
decided that the release of such concentrates be granted.
Collector of Customs, Ang-Angco, acted while he is still in
doubt and authorized the release of the concentrates upon
payment of the corresponding duties, customs charges, fees
and taxes. When Commissioner of Customs Manuel P.
Manahan learned of the release of the concentrates in
question, he immediately ordered their seizure but only a
negligible portion thereof remained in the warehouse.
Whereupon, he filed an administrative complaint against
Collector of Customs Ang-Angco charging him with having
committed a grave neglect of duty and observed a conduct
prejudicial to the best interest of the customs service.
Issue/s (based sa case):
W/N Executive Secretary Natalio P. Castillo in acting on his
case by authority of the President (in the sense of considering
Ang-Angco as resigned from notice thereof), violated the
guaranty vouchsafed by the Constitution to officers and
employees in the classified service in that he acted in
violation of Section 16 (i), Sec 18 (b) and Sec 32 of the
Civil Service Act of 1959, and of Section 4, Article XII of
our Constitution
Main Issue: W/N the President has the power to take direct
action on the case of petitioner even if he belongs to the
classified service in spite of the provisions now in force in
the Civil Service Act of 1959
(in other words, does the executive's power of control covers
Ang-Angco's situation)*
*Note/Explanation
(Ang-Angco's contention): petitioner is an officer who belongs to the classified civil service and
is not a presidential appointee, but one appointed by the Secretary of Finance under the
Revised Administrative Code, thus believes that he cannot be removed from the service by the
President in utter disregard of the provisions of the Civil Service Act of 1959.
(Contention ng respondents): But it is their theory that the pertinent provisions of the Civil
Service Law applicable to employees in the classified service do not apply to the particular
case of petitioner since to hold otherwise would be to deprive the President of his power of
control over the officers and employees of the executive branch of the government. In other
words, respondents contend that, whether the officers or employees concerned are
presidential appointees or belong to the classified service, if they are all officers and
employees in the executive department, they all come under the control of the President and,
therefore, his power of removal may be exercised over them directly without distinction.

Ruling:
The power of Executive Control may extend to the power to
investigate, suspend or remove officers and employees who
belong under the executive department IF they are presidential
appointees but not with regard to those under the classified
service.
Even granting for administrative purposes, the President of the
Philippines is considered as the Department Head of the Civil
Service Commission, his power to remove is still subject to the
Civil Service Act of 1959, and we already know with regard to
officers and employees who belong to classified service the
finality of the action is given to the Commissioner of Civil
Service or the Civil Board of Appeals.
This is in line with Art VII Sec 10 (3) of the Constitution. The
Congress has provided by law a procedure for their removal
(i.e. Civil Service Act of 1959) thus Ang-Angco's situation is not
covered by the Executive's power to remove a person from the
office.
WHEREFORE, it is hereby ordered that petitioner be
immediately reinstated to his office as Collector of Customs for
the Port of Manila, without prejudice of submitting his case to
the Commissioner of Civil Service to be dealt with in
accordance with law. No costs.
NAMARCO v. Arca (1969)
Facts:
Respondent Juan T. Arive was the Manager of the TrafficStorage Department of the NAMARCO was found guilty of the
charges (violating Management Memorandum Order dated
February 1, 1960, directing "that the allocation and deliveries of
merchandise imported under the so-called Trade Assistance
Program to its designated beneficiaries be stopped;" and
causing the improper release of shipments intended for
delivery upon full payment thereof by the Federation of United
NAMARCO Distributors (FUND)).
Subsequently, the General Manager issued Administrative
Order No. 137, series of 1960, holding Arive guilty of the
charges and dismissing him from the service. On November 4.
1960, the Board of Directors adopted Resolution No. 584-60
dismissing Arive from the service effective as of the date of his
suspension, with prejudice to his reinstatement in the
NAMARCO and to all benefits to which he would otherwise
have been entitled, Arive filed a motion for reconsideration,
which was denied.
The NAMARCO was advised by the Office of the President of
the appeal, and was asked to forward the records of the
administrative case. On January 26, 1965, then Executive
Secretary Ramon A. Diaz, presumably acting for the President,
handed down a decision setting aside Resolution No. 584-60
of the NAMARCO and reinstating Juan T. Arive to his former
position.
In the decision it was pointed out that the order of the
NAMARCO stopping the further delivery of commodities
imported under the trade assistance program to the
designated beneficiaries had been subsequently declared
illegal by the Supreme Court, on the ground that said
order was a violation of the contract of sale; hence, it

would not be proper to hold Arive administratively liable


for his failure to comply with said order; and that the Pasig
River Bodegas being private warehouses over which Arive
did not have supervision, much less control, the release of
the commodities therefrom could have been effected even
had Arive tried to block it.
On April 6, 1965, the NAMARCO, through its General
Manager, in a letter addressed to the President, asked for a
reconsideration of the decision ordering Arive's reinstatement.
In that letter it was contended that the Office of the
President had no jurisdiction to review any decision of the
NAMARCO Board of Directors removing, suspending, or
otherwise disciplining any of its subordinate employees*
In a letter to the NAMARCO dated June 8, 1965, then
Executive Secretary Ramon A. Diaz, this time expressly acting
"[b]y authority of the President," refused to reconsider the
decision, stating that the President had jurisdiction under his
constitutional power of control over all executive departments,
bureaus and offices, and directing that the decision be
implemented.
The NAMARCO filed a second motion for reconsideration; and
on November 17, 1965, the President, through Salvador
Marino, as Acting Executive Secretary, denied the motion and
again directed immediate compliance with the order of
reinstatement. On December 9, 1965, the Office of the
President, acting on complaints of Arive that he had not been
reinstated in spite of the denial of the NAMARCO's two
motions for reconsideration, sent a telegram to the General
Manager requesting him to act on the case and to comment
within forty-eight hours; but the said General Manager neither
acted on the case nor commented.
On December 23, 1965, respondent Juan T. Arive filed a
complaint (Civil Case No. 63720) with the Court of First
Instance of Manila against the NAMARCO and the members of
its Board of Directors for reinstatement and damages, with
prayer for a writ of preliminary mandatory injunction.
Judge issued an order dated January 12, 1966, the pertinent
portion of which reads:
... The Court is, however, of the view that the President of
the Philippines does not only exercise supervision but
also control over all government-owned and controlled
corporations including the NAMARCO; hence, he may
review, revise, alter, modify or nullify the decision or
action of the Board of Directors of any government-owned
and controlled corporation and substitute his judgment for
that of the latter. Plaintiff's right to reinstatement, therefore,
appears to be very clear; and considering that the effect of the
issuance of the writ prayed for is rather to re-establish and
maintain a pre-existing continuing relation between the parties
and considering further that there is an invasion of plaintiff's
right and the injury is a continuing one, the Court hereby grants
plaintiff's prayer and hereby orders the issuance of a writ of
preliminary mandatory injunction directing the defendants to
immediately reinstate the plaintiff to his position as Manager of
the Traffic Storage Department of the National Marketing
Corporation upon filing a bond in the amount of P5,000.00.
Defendants filed a motion for reconsideration; and when the
motion was denied, they filed the present petition with this
Court, which on March 15, 1966, issued a writ of preliminary
injunction.
-----* Petitioners contend that the word "offices," interpreted in the
light of the preceding words "executive departments," and
"bureaus," refers to offices performing governmental functions

which have no juridical personality, and, therefore, does not


include government-owned and controlled corporations (such
as NAMARCO)
Respondents maintain that he had, and they anchor their stand
on Section 10(1), Article VII, of the Constitution
(in short ang makukuha sa facts e yung decision na dismissal
ni Arive dahil naviolate nya daw yung Management
Memorandum Order, e hindi sinuportahan ng Executive
branch. Tapos dahil dun, since gusto ireinstate ng executive si
Arive, pinaglalaban naman ng NAMARCO na hindi sila sakop
ng power ng president at dapat yung charter nila ang
masunod. Yun lang talaga yan, kaso di ko na lang tinanggal
yung details baka sakali masyadong magulo pag sobrang iksi
ng nilagay ko)

Issue:
W/N the President has the authority to reverse the decision of
the Board of Directors of the NAMARCO and to order the
reinstatement of Juan T. Arive
Ruling:
The President of the Philippines authority to review and
reverse the decision of the NAMARCO Board of Directors
dismissing Juan T. Arive from his position in the NAMARCO
and to order his reinstatement falls within the constitutional
power of the President over all executive departments,
bureaus and offices. Under our governmental set-up,
corporations owned or controlled by the government, such as
the NAMARCO, partake of the nature of government bureaus
or offices, which are administratively supervised by the
Administrator of the Office of Economic Coordination, whose
compensation and rank shall be that of a head of an Executive
Department and who shall be responsible to the President of
the Philippines under whose control his functions ... shall be
exercised. (Executive Order No. 386 of December 22, 1950,
section 1, issued under the Reorganization Act of 1950).
The fact that section 13(d) of Republic Act No. 1345 (the
NAMARCO Charter and likewise section 11(d) of the Uniform
Charter for Government Owned or Controlled Corporations
(Ex. Order No. 399 of January 5, 1951) which authorize the
general manager of such corporations, with the approval of the
Board of Directors, to remove for cause any subordinate
employee of the Corporation do not provide for an appeal from
the general managers decision of removal to any superior
officer, body or agency, does not mean that no appeal lies from
such decision to the President.
Arive's right to reinstatement by virtue of the President's
decision, which was reiterated twice in denying the petitioner's
persistent motions for reconsideration was, therefore, clearly
established, and which is now final and binding upon
petitioners, and respondent judge did not act without
jurisdiction or with grave abuse of discretion in issuing the writ
of preliminary mandatory injunction for his immediate
reinstatement. We deem it unnecessary to pass upon the other
issues raised by the parties, which are after all, merely
incidental to the main issue of the President's authority to
review and reverse Resolution No. 584-60 of the NAMARCO
Board of Directors.
WHEREFORE, the petition is dismissed, with costs against
petitioners.1aw
The writ of preliminary injunction issued on March 15, 1966
against the enforcement of respondent judge's order dated

January 12, 1966 and writ of preliminary mandatory injunction


dated January 14, 1966 in Civil Case No. 63720 of the Court of
First Instance of Manila is hereby dissolved effective
immediately.
Drilon v. Lim 235 SCRA 135, 1994
Facts:
The principal issue on this case is the constitutionality of
Section 187 of the Local Government Code.
Procedure For Approval And Effectivity Of Tax Ordinances And
Revenue Measures; Mandatory Public Hearings. The
procedure for approval of local tax ordinances and revenue
measures shall be in accordance with the provisions of this
Code: Provided, That public hearings shall be conducted for
the purpose prior to the enactment thereof; Provided, further,
That any question on the constitutionality or legality of tax
ordinances or revenue measures may be raised on appeal
within thirty (30) days from the effectivity thereof to the
Secretary of Justice who shall render a decision within sixty
(60) days from the date of receipt of the appeal: Provided,
however, That such appeal shall not have the effect of
suspending the effectivity of the ordinance and the accrual and
payment of the tax, fee, or charge levied therein: Provided,
finally, That within thirty (30) days after receipt of the decision
or the lapse of the sixty-day period without the Secretary of
Justice acting upon the appeal, the aggrieved party may file
appropriate proceedings with a court of competent jurisdiction.
Secretary of Justice Franklin M. Drilon, on appeal of four oil
companies and a taxpayer, declared Ordinance No. 7794 or
Manila Revenue Code null and void for non-compliance with
the procedures for enactment of the ordinance and for
containing certain provisions contrary to law.
The City of Manila filed a petition for certiorari where the
Regional Trial Court overruled the resolution of the Secretary
of Justice and held the ordinance inter alia that the procedures
have been observed. Further, it declared the Section 187 of the
Local Government Code as unconstitutional on the point that it
vested the power of control over local governments on the
Secretary of Justice as that control violated the local autonomy.
Issue:
Whether or not Section 187 of the Local Government Code is
unconstitutional
Held:
The court affirmed the trial courts decision on declaring the
Ordinance No. 7794 valid and had observed the required
procedures for its enactment but reversed the judgment on the
unconstitutionality of the Section 187 of the Local Government
Code.
The Court explained the difference between control and
supervision.
An officer in control lays down the rules in the doing of an act.
If they are not followed, he may, in his discretion, order the act
undone or re-done by his subordinate or he may even decide
to do it himself. Supervision does not cover such authority. The
supervisor or superintendent merely sees to it that the rules
are followed, but he himself does not lay down such rules, nor
does he have the discretion to modify or replace them. If the
rules are not observed, he may order the work done or re-done
but only to conform to the prescribed rules. He may not
prescribe his own manner for the doing of the act. He has no

judgment on this matter except to see to it that the rules are


followed.
Section 187 authorizes the Secretary of Justice to review only
the constitutionality or legality of the tax ordinance and, if
warranted, to revoke it on either or both these grounds. When
he alters or modifies or sets aside a tax ordinance, he is not
also permitted to substitute his own judgment for the judgment
of the local government that enacted the measure. Secretary
Drilon did set aside the Manila Revenue Code, but he did not
replace it with his own version of what the Code should be.
What he found only was that it was illegal. All he did in
reviewing the said measure was determine if the petitioners
were performing their functions in accordance with law, that is,
with the prescribed procedure for the enactment of tax
ordinances and the grant of powers to the city government
under the Local Government Code. As we see it, that was an
act not of control but of mere supervision.
Joson v. Torres 290 SCRA 279, 1998
Facts:
Morning of September 12, 1996, private respondents Oscar C.
Tinio,the Vice-Governor of Nueva Ecija with Loreto P.
Pangilinan, Crispulo S. Esguerra, Solita C. Santos, Vicente C.
Palilio and Napoleon G. Interior as members of the
SangguniangPanlalawigan were in the session hall of the
provincial capitol for a scheduled session of the
SangguniangPanlalawigan when the petitioner, Eduardo
NonatoJoson, the governor of the province of Nueva Ecija
barged in the hall with armed men and uttered threatening
words against them. Respondents claimed that the incident
was because of the pending legislative measure supported by
the petitioner where the province obtain a loan of 150 million
from the Philippine National Bank but no session by the
SangguniangPanlalawigan was held that day for the lack of
quorum and the proposed legislative measure was not
considered. The respondents disapproved of the loan because
the province of Nueva Ecija still had an unliquidated obligation
of no more that 70 million incurred without prior authorization
by the SangguniangPanlalawigan and the provincial budget
officer and treasurer disclosed that the province could not
afford to have another obligation.
A letter-complaint was submitted with the joint affidavit of
Elnora Escombien and Jacqueline Jane Perez, two (2)
employees of the SangguniangPanlalawigan who witnessed
the incident.
They were thus summoned by the DILG to a settlement but did
not comply so they were asked to give answers with regards to
the non-settlement where, Joson failed to do so and so was
declared of waiving his right. Joson filed motion to dismiss
alleging that the letter of complaint was not verified on the day
it was filed and hat DILG had no jurisdiction over the case and
no authority to require him to answer. Executive Secretary
Torres issued an order, by authority of President, placing
petitioner Joson on a 60 days suspension pending
investigation on the charges against him.
On January 8, 1998, the Executive Secretary, by authority of
the President, adopted the findings and recommendation of the
DILG Secretary. He imposed on petitioner the penalty of
suspension from office for six (6) months without pay, to wit:
"WHEREFORE, as recommended by the Secretary of the
Interior and Local Government, respondent Nueva Ecija
Governor Eduardo NonatoJoson is hereby found guilty of the
offenses charged and is meted the penalty of suspension from
office for a period of six (6) months without pay."

Issue:
Whether or not the DILG secretary has jurisdiction and
authority on the case
Held:
The Court held that the resolution of January 8, 1998 by the
executive secretary was null and void but affirmed the
preventive sixty(60) days suspension of the petitioner. The
power of the President over administrative disciplinary cases
against elective local officials is derived from his power of
general supervision over local governments. Section 4, Article
X of the 1987 Constitution provides:
"Sec. 4. The President of the Philippines shall exercise general
supervision over local governments. Provinces with respect to
component cities and municipalities, and cities and
municipalities with respect to component barangays shall
ensure that the acts of their component units are within the
scope of their prescribed powers and functions."
Supervision means the authority to see that the subordinate
officers perform their duties. An official may take action
prescribed by law to make them do their duties. As discipline is
tantamount to supervision, taking disciplinary actions is
authorized to ensure that the laws and public service is met
faithfully. Where the president is the disciplining authority, it
was also expressed in A.O. No. 23 that the president may act
through the Executive Secretary. The power to discipline
includes the power to investigate and section 17 of Article 7
states that control of the president on executive departments,
bureaus and offices. Thus, validating the actions performed by
the Executive Secretary and Secretary of Interior and Local
Governments.
PASEI v. Torres-225 SCRA 417
Facts:
On June 1, 1991, Department of Labor and Employment
Secretary Ruben D. Torres issued Department Order No. 16,
series of 1991, temporarily suspending the private recruitment
of Filipino workers going to Hongkong. This is a result of the
published stories regarding abuse suffered by housemaids in
Hongkong. The DOLE, through the POEA took over the
business of deploying such Hong Kong-bound workers.
In regards to the issued circular by the DOLE, Philippine
Overseas Employment Administration issued Memorandum
Circular No. 30, series of 1991 on July 10, 1991, providing
Guidelines on the Government processing and deployment of
Filipino domestic helpers to Hong Kong and the accreditation
of Hong Kong recruitment agencies who intend to hire Filipino
domestic helpers.
Then on August 1, 1991, Memorandum Circular No. 37, series
of 1991, was issued by the administrator of POEA regarding
the processing of contracts of Filipinos working for Hongkong.
The petitioner, Philippine Association of Service Exporters, Inc.
filed a petition on September 2, 1991 for prohibition to annul
the issued circulars of DOLE and POEA and to prohibit their
implementation.
Issue/s:
Whether or not the respondents acted with grave abuse of
discretion and/or in excess of their rule-making authority in
issuing the circulars
Whether or not the assailed DOLE and POEA circulars are
contrary to the Constitution, are unreasonable, unfair and
oppress

Held:
The Court held that the respondents have the power to issue
the circulars and those circulars are a valid exercise of the
police power as delegated by to the executive branch of the
government but the issuance of the said circulars are defective
and unenforceable as it lacks the legality of publication and
filing in the National Administrative Registrar as required by the
law,
Article 36 of the Labor Code grants the Labor Secretary the
power to restrict and regulate recruitment and placement
activities.
Further, administrative bodies like the DOLE and POEA who
has quasi-legislative and quasi-judicial character is necessary
to expertly deal and expeditiously to help regulate the activities
of the society at present. Moreover:
The power to "restrict and regulate conferred by Article 36 of
the Labor Code involves a grant of police power (City of Naga
vs. Court of Appeals, 24 SCRA 898). To "restrict" means "to
confine, limit or stop" (p. 62, Rollo) and whereas the power to
"regulate" means "the power to protect, foster, promote,
preserve, and control with due regard for the interests, first and
foremost, of the public, then of the utility and of its
patrons" (Philippine Communications Satellite Corporation vs.
Alcuaz, 180 SCRA 218).
The circulars are intended to curtail, not to stop the recruitment
of domestic workers who may be abused by private
recruitment firms thus, making it reasonable as it is intended
for the general welfare of the Filipino Domestic workers.

You might also like